You are on page 1of 37

Arrhythmia Tutorial 2 Tachycardia

1. A 32 year old woman presents with


palpitations. EKG 1a is obtained. She is
treated with adenosine, and EKG 1b is
obtained. What is the most likely cause of
this tachycardia?
A. AV reentry using an accessory pathway
B. AV nodal reentrant tachycardia
C. Atrial tachycardia
D. Sinus tachycardia
1a
1b
2. A 47 year old female with a history of alcoholism
and schizophrenia is admitted to the hospital with
pyelonephritis and started on intravenous
ciprofloxacin. She complains of palpitations and
one episode of near syncope. EKG 2 is obtained.
Which of the following therapies is most
appropriate at this point?
A. Temporary pacing
B. Isoproterenol
C. Potassium and magnesium repletion
D. Change from intravenous to oral ciprofloxacin
E. Atropine
2.
3. A 50 year old smoker with chronic bronchitis
treated with beta-2 agonists and theophylline
presents with dyspnea. His temperature is 102 F, BP
130/70 mm Hg, and RR is 32 breaths/min; pulse is
irregular. There are diffuse wheezes, the nail beds
are cyanotic, and the jugular veins are moderately
distended. EKG 10 is obtained. What is the EKG
diagnosis? Which intervention is most appropriate?
A. Digitalization
B. Electrical cardioversion
C. Oxygen, acetaminophen, and
discontinuation of theophylline
D. Activated charcoal
E. Verapamil, 1 mg/min IV
3
4a. A 60 year old man presents to the emergency
room with 3 mm of ST segment elevation in
leads V
1
to V
3
. His blood pressure is 120/80 mm
Hg, and he is complaining of chest discomfort
and is diaphoretic. The rhythm shown in EKG 4a
is noted. What is the EKG diagnosis? What
immediate intervention is most appropriate?
A. Carotid sinus massage
B. Intravenous beta blockade
C. Intravenous amiodarone
D. Electrical cardioversion
E. Intravenous procainamide
4a.
4b. Sinus rhythm is restored, he is given
thrombolytic therapy, bolused with 1 mg/kg
lidocaine, and started on a lidocaine
infusion at 2 mg/min. The rhythm shown in
EKG 4b is noted. What is the EKG
diagnosis? Which intervention is most
appropriate?
A. Discontinue the lidocaine
B. Give additional lidocaine, 0.5 mg/kg IV
bolus, and increase the lidocaine to 3 mg/ml
C. Change the lidocaine to amiodarone
D. Electrical cardioversion
E. Observation
4b
5. A 69 year old man has been undergoing evaluation for
palpitations. His resting EKG is EKG 5a. A previous
echocardiogram and exercise stress test have been
normal. He presents to the emergency room with
palpitations, and EKG 5b is obtained. On 12-lead
monitoring, the rhythm shown in EKG 5c is noted. Which
of the following is true?
A. This condition rarely results in sudden death
B. This arrhythmia is sensitive to adenosine
C. This arrhythmia usually responds to calcium
channel blockers and beta blockers
D. This arrhythmia is amenable to catheter
ablation
E. All of the above
5a.
5b.
5c.
6. A 55 y/o male with history of alcoholism is
pulled out of Lake Michigan after a near-
drowning. The following ECG suggests:

A. Hyperkalemia
B. Acute myocardial infarction
C. Hypothermia
D. Left bundle branch block
6.
7. A 64 year old woman with a history of
previous myocardial infarction is admitted
for chest pain, and noted to have the rhythm
seen in EKG 7. Which of the following is the
most appropriate treatment?
A. Intravenous amiodarone
B. Intravenous lidocaine
C. Beta blockade
D. Intravenous diltiazem
E. An electrophysiology consult for implantable
defibrillator placement
7
8A. A 74 year old man with a previous
history of MI presents with palpitations. EKG
8a is obtained. What is the EKG diagnosis?
A. Atrial fibrillation
B. Ventricular tachycardia
C. Junctional rhythm with premature ventricular
contractions
D. Sinus rhythm with evidence of acute
myocardial infarction
8a.
8B. He has recurrent symptoms, and EKG 8b
is obtained. What is the rhythm?
A. Atrial fibrillation
B. Ventricular tachycardia
C. Junctional rhythm with premature ventricular
contractions
D. Sinus rhythm with evidence of acute
myocardial infarction
8b.
9. A 53 year old with no past medical history
presents with palpitations and EKG 9. His blood
pressure is 105/70 mm Hg, and an
echocardiogram reveals normal left ventricular
function and wall motion. The most appropriate
therapy at this point would be:
A. Adenosine for AV node reentrant
tachycardia with RBBB
B. Emergent cardioversion without sedation
C. Intravenous diltiazem for atrial flutter
D. Intravenous amiodarone
9.
10A. A 27 year old man with no previous
medical history presents with palpitations
and EKG 10a. His blood pressure is 100/60
mm Hg. Which of the following is correct?
A. IV diltiazem should be started for rate control of
supraventricular tachycardia
B. IV amiodarone should be started for stable
ventricular tachycardia
C. IV diltiazem should be given for right ventricular
outflow tract tachycardia
D. IV amiodarone should be given for atrial
fibrillation with WPW syndrome
E. Digoxin should be given for rate control of atrial
fibrillation
10a.
10b. The patient becomes diaphoretic, and
blood pressure drops to 70/40 mm Hg. What
is the appropriate next step?
A. IV amiodarone
B. IV procainamide
C. IV lidocaine
D. IV diltiazem
E. Electrical cardioversion
10c. The patient is converted to sinus
rhythm. EKG 10c is obtained. Which of the
following is correct?
A. The patient is not at increased risk for
sudden death
B. The patient should undergo catheter
ablation
C. The patient should have an implantable
defibrillator placed
D. The patient should have an ICD and a
catheter ablation
10c.
10d. The patient has recurrent symptoms
and EKG 10d is obtained. What is the most
likely etiology of the rhythm?
A. Orthodromic reentrant tachycardia (AV
reentrant tachycardia)
B. AV nodal reentry tachycardia
C. Atrial fibrillation
D. Atrial tachycardia
10d.
11. A 66 year old with a pacemaker presents
with palpitations. Blood pressure is 100/70,
and lungs are clear. EKG 11 is obtained. The
best course of action at this point would be
to:
A. Give diltiazem 10 mg IV
B. Give lidocaine 100 mg IV
C. Put a magnet over the chest
D. Sedate and perform electrical cardioversion
E. Give amiodarone 150 mg IV
11.
12. A 57 year old woman with history of
laxative abuse and recent vomiting has a
seizure at home and is transferred to the
emergency department. EKG 12 is obtained.
The nurse asks which antiarrhythmic agent
you wish to choose. What is the best answer?
A. Amiodarone
B. Procainamide
C. Flecainide
D. Diltiazem
E. None of the above
12.

You might also like